K
Khách

Hãy nhập câu hỏi của bạn vào đây, nếu là tài khoản VIP, bạn sẽ được ưu tiên trả lời.

NV
2 tháng 7 2020

\(\sum\frac{a}{a^2+1+2b+2}\le\sum\frac{a}{2a+2b+2}\)

Nên ta chỉ cần chứng minh: \(\sum\frac{a}{2a+2b+2}\le\frac{1}{2}\Leftrightarrow\sum\frac{a}{a+b+1}\le1\)

\(\Leftrightarrow\sum\frac{b+1}{a+b+1}\ge2\)

Đặt \(P=\sum\frac{b+1}{a+b+1}=\sum\frac{\left(b+1\right)^2}{\left(b+1\right)\left(a+b+1\right)}=\sum\frac{\left(b+1\right)^2}{ab+a+b^2+2b+1}\)

\(P\ge\frac{\left(a+b+c+3\right)^2}{ab+a+b^2+2b+1+bc+b+c^2+2c+1+ca+c+a^2+2a+1}\)

\(P\ge\frac{a^2+b^2+c^2+2\left(ab+bc+ca\right)+6\left(a+b+c\right)+9}{a^2+b^2+c^2+ab+bc+ca+3\left(a+b+c\right)+3}\)

\(P\ge\frac{2\left(ab+bc+ca\right)+6\left(a+b+c\right)+12}{ab+bc+ca+3\left(a+b+c\right)+6}=2\) (đpcm)

Dấu "=" xảy rakhi \(a=b=c=1\)

18 tháng 5 2018

ta có:

\(\dfrac{a}{a^2+2b+3}=\dfrac{a}{a^2+1+2b+2}\le\dfrac{a}{2\left(a+b+1\right)}\)

tương tự như vậy ta chứng minh được các bđt tương tự, sau đó, cộng lại, ta được:

\(P\le\dfrac{1}{2}\left(\dfrac{a}{a+b+1}+\dfrac{b}{b+c+1}+\dfrac{c}{c+a+1}\right)\)

áp dụng bđt bunhiacoxki. ta có:

\(\dfrac{a}{a+b+1}=\dfrac{a\left(a+b+c^2\right)}{\left(a+b+1\right)\left(a+b+c^2\right)}\le\dfrac{a\left(a+b+c^2\right)}{\left(a^2+b^2+c^2\right)^2}=\dfrac{a^2+ab+ac^2}{9}\)

tương tự như vậy ta chứng minh được các bđt tương tự, sau đó, cộng lại, ta được:

\(P\le\dfrac{1}{18}\left(\left(a^2+b^2+c^2\right)+\left(ab+bc+ca\right)+\left(ac^2+ba^2+cb^2\right)\right)\)

ta có:

\(ab+bc+ca\le a^2+b^2+c^2=3\)

\(\left(a+b+c\right)^2\le3\left(a^2+b^2+c^2\right)=3.3=9\Rightarrow a+b+c\le3\)

\(ac^2+ba^2+cb^2=\sqrt{\left(a.c^2+b.a^2+c.b^2\right)^2}\le\sqrt{\left(a+b+c\right)\left(a^2+b^2+c^2\right)}\le\sqrt{3.3}=3\)

\(\Rightarrow P\le\dfrac{1}{18}\left(3+3+3\right)=\dfrac{1}{2}\)

Dấu "=" xảy ra khi a=b=c=1

18 tháng 11 2019

1. Vai trò a, b, c như nhau. Không mất tính tổng quát. Giả sử \(a\ge b\ge0\)

\(ab+bc+ca=3\). Do đó \(ab\ge1\)

Ta cần chứng minh rằng \(\frac{1}{1+a^2}+\frac{1}{1+b^2}\ge\frac{2}{1+ab}\left(1\right)\)

\(\frac{2}{1+ab}+\frac{1}{1+c^2}\ge\frac{3}{2}\left(2\right)\)

Thật vậy: \(\left(1\right)\Leftrightarrow\frac{1}{1+a^2}-\frac{1}{1+ab}+\frac{1}{1+b^2}-\frac{1}{1+ab}\ge0\\ \Leftrightarrow\left(ab-a^2\right)\left(1+b^2\right)+\left(ab-b^2\right)\left(1+a^2\right)\ge0\\ \Leftrightarrow\left(a-b\right)\left[-a\left(1+b^2\right)+b\left(1+a^2\right)\right]\ge0\\ \Leftrightarrow\left(a-b\right)^2\left(ab-1\right)\ge0\left(BĐT:đúng\right)\)

\(\left(2\right)\Leftrightarrow c^2+3-ab\ge3abc^2\\ \Leftrightarrow c^2+ca+bc\ge3abc^2\Leftrightarrow a+b+c\ge3abc\)

BĐT đúng, vì \(\left(a+b+c\right)^2>3\left(ab+bc+ca\right)=q\)

\(ab+bc+ca\ge3\sqrt[3]{\left(abc\right)^2}\)

Nên \(a+b+c\ge3\ge3abc\)

Từ (1) và (2) ta có \(\frac{1}{1+a^2}+\frac{1}{1+b^2}+\frac{1}{1+c^2}\ge\frac{3}{2}\)

Dấu ''='' xảy ra \(\Leftrightarrow a=b=c=1\)

18 tháng 11 2019

Áp dụng BĐT Cauchy dạng \(\frac{9}{x+y+z}\le\frac{1}{x}+\frac{1}{y}+\frac{1}{z}\), ta được

\(\frac{9}{a+3b+2c}=\frac{1}{a+c+b+c+2b}\le\frac{1}{9}\left(\frac{1}{a+c}+\frac{1}{b+c}+\frac{1}{2b}\right)\)

Do đó ta được

\(\frac{ab}{a+3b+2c}\le\frac{ab}{9}\left(\frac{1}{a+c}+\frac{1}{b+c}+\frac{1}{2b}\right)=\frac{1}{9}\left(\frac{ab}{a+c}+\frac{ab}{b+c}+\frac{a}{2}\right)\)

Hoàn toàn tương tự ta được

\(\frac{bc}{2a+b+3c}\le\frac{1}{9}\left(\frac{bc}{a+b}+\frac{bc}{b+c}+\frac{b}{2}\right);\frac{ac}{3a+2b+c}\le\frac{1}{9}\left(\frac{ac}{a+b}+\frac{ac}{b+c}+\frac{c}{2}\right)\)

Cộng theo vế các BĐT trên ta được

\(\frac{ab}{a+3b+2c}+\frac{bc}{b+3c+2a}+\frac{ca}{c+3a+2b}\le\frac{1}{9}\left(\frac{ac+bc}{a+b}+\frac{ab+ac}{b+c}+\frac{bc+ab}{a+c}+\frac{a+b+c}{2}\right)=\frac{a+b+c}{6}\)Vậy BĐT đc CM

ĐẲng thức xảy ra khi và chỉ khi a = b = c >0

11 tháng 11 2019

2/ Không mất tính tổng quát, giả sử \(c=min\left\{a,b,c\right\}\).

Nếu abc = 0 thì có ít nhất một số bằng 0. Giả sử c = 0. BĐT quy về: \(a^2+b^2\ge2ab\Leftrightarrow\left(a-b\right)^2\ge0\) (luôn đúng)

Đẳng thức xảy ra khi a = b; c = 0.

Nếu \(abc\ne0\). Chia hai vế của BĐT cho \(\sqrt[3]{\left(abc\right)^2}\)

BĐT quy về: \(\Sigma_{cyc}\sqrt[3]{\frac{a^4}{b^2c^2}}+3\ge2\Sigma_{cyc}\sqrt[3]{\frac{ab}{c^2}}\)

Đặt \(\sqrt[3]{\frac{a^2}{bc}}=x;\sqrt[3]{\frac{b^2}{ca}}=y;\sqrt[3]{\frac{c^2}{ab}}=z\Rightarrow xyz=1\)

Cần chúng minh: \(x^2+y^2+z^2+3\ge2\left(\frac{1}{x}+\frac{1}{y}+\frac{1}{z}\right)\)

\(\Leftrightarrow x^2+y^2+z^2+2xyz+1\ge2\left(xy+yz+zx\right)\) (1)

Theo nguyên lí Dirichlet thì trong 3 số x - 1, y - 1, z - 1 tồn tại ít nhất 2 số có tích không âm. Không mất tính tổng quát, giả sử \(\left(x-1\right)\left(y-1\right)\ge0\)

\(\Rightarrow2xyz\ge2xz+2yz-2z\). Thay vào (1):

\(VT\ge x^2+y^2+z^2+2xz+2yz-2z+1\)

\(=\left(x-y\right)^2+\left(z-1\right)^2+2xy+2xz+2yz\)

\(\ge2\left(xy+yz+zx\right)\)

Vậy (1) đúng. BĐT đã được chứng minh.

Đẳng thức xảy ra khi a = b = c hoặc a = b, c = 0 và các hoán vị.

Check giúp em vs @Nguyễn Việt Lâm, bài dài quá:(

6 tháng 7 2020

Để đưa về chứng minh $(1)$ và $(2)$ ta dùng:

Định lí SOS: Nếu \(X+Y+Z=0\) thì \(AX^2+BY^2+CZ^2\ge0\)

khi \(\left\{{}\begin{matrix}A+B+C\ge0\\AB+BC+CA\ge0\end{matrix}\right.\)

Chứng minh: Vì \(\sum\left(A+C\right)=2\left(A+B+C\right)\ge0\)

Nên ta có thể giả sử \(A+C\ge0\). Mà $X+Y+Z=0$ nên$:$

\(AX^2+BY^2+CZ^2=AX^2+BY^2+C\left[-\left(X+Y\right)\right]^2\)

\(={\frac { \left( AX+CX+CY \right) ^{2}}{A+C}}+{\frac {{Y}^{2} \left( AB+AC+BC \right) }{A+C}} \geq 0\)

NV
19 tháng 6 2019

a/ BĐT sai, cho \(a=b=c=2\) là thấy

b/ \(VT=\frac{a^4}{a^2+2ab}+\frac{b^4}{b^2+2bc}+\frac{c^4}{c^2+2ac}\ge\frac{\left(a^2+b^2+c^2\right)^2}{\left(a+b+c\right)^2}=\frac{\left(a^2+b^2+c^2\right)\left(a^2+b^2+c^2\right)}{\left(a+b+c\right)^2}\)

\(VT\ge\frac{\left(a^2+b^2+c^2\right)\left(a+b+c\right)^2}{3\left(a+b+c\right)^2}=\frac{1}{3}\left(a^2+b^2+c^2\right)\)

Dấu "=" xảy ra khi \(a=b=c\)

c/ Tiếp tục sai nữa, vế phải là \(\frac{3}{2}\) chứ ko phải \(2\), và hy vọng rằng a;b;c dương

\(VT=\frac{a^2}{abc.b+a}+\frac{b^2}{abc.c+b}+\frac{c^2}{abc.a+c}\ge\frac{\left(a+b+c\right)^2}{abc\left(a+b+c\right)+a+b+c}\)

\(VT\ge\frac{9}{3abc+3}\ge\frac{9}{\frac{3\left(a+b+c\right)^3}{27}+3}=\frac{9}{\frac{3.3^3}{27}+3}=\frac{9}{6}=\frac{3}{2}\)

Dấu "=" xảy ra khi \(a=b=c=1\)

NV
23 tháng 6 2019

Ta có:

\(a^3+b^3+b^3\ge3ab^2\) ; \(b^3+c^3+c^3\ge3bc^2\) ; \(c^3+a^3+a^3\ge3ca^2\)

Cộng vế với vế \(\Rightarrow a^3+b^3+c^3\ge ab^2+bc^2+ca^2\)

\(\frac{a^5}{b^2}+\frac{b^5}{c^2}+\frac{c^5}{a^2}=\frac{a^6}{ab^2}+\frac{b^6}{bc^2}+\frac{c^6}{ca^2}\ge\frac{\left(a^3+b^3+c^3\right)^2}{ab^2+bc^2+ca^2}\ge\frac{\left(a^3+b^3+c^3\right)^2}{a^3+b^3+c^3}=a^3+b^3+c^3\)

30 tháng 5 2020

Ta có: \(a^2+2b+3=\left(a^2+1\right)+2\left(b+1\right)\ge2\left(a+b+1\right)\)

Tương tự ta có: \(b^2+2c+3\ge2\left(b+c+1\right)\)\(c^2+2a+3\ge2\left(c+a+1\right)\)

Từ đó suy ra\(\frac{a}{a^2+2b+3}+\frac{b}{b^2+2c+3}+\frac{c}{c^2+2a+3}\)\(\le\frac{a}{2\left(a+b+1\right)}+\frac{b}{2\left(b+c+1\right)}+\frac{c}{2\left(c+a+1\right)}\)

\(=\frac{1}{2}\left(\frac{a}{a+b+1}+\frac{b}{b+c+1}+\frac{c}{c+a+1}\right)\)

Đặt \(K=\frac{a}{a+b+1}+\frac{b}{b+c+1}+\frac{c}{c+a+1}\), ta đi chứng minh \(K\le1\)

Thật vậy: \(3-K=\frac{b+1}{a+b+1}+\frac{c+1}{b+c+1}+\frac{a+1}{c+a+1}\)

\(=\frac{\left(b+1\right)^2}{\left(b+1\right)\left(a+b+1\right)}+\frac{\left(c+1\right)^2}{\left(c+1\right)\left(b+c+1\right)}+\frac{\left(a+1\right)^2}{\left(a+1\right)\left(c+a+1\right)}\)

\(\ge\frac{\left(a+b+c+3\right)^2}{\left(b+1\right)\left(a+b+1\right)+\left(c+1\right)\left(b+c+1\right)+\left(a+1\right)\left(c+a+1\right)}\)(*)

Ta có: \(\left(b+1\right)\left(a+b+1\right)+\left(c+1\right)\left(b+c+1\right)+\left(a+1\right)\left(c+a+1\right)\)\(=3\left(a+b+c\right)+ab+bc+ca+a^2+b^2+c^2+3\)

(Mình gõ bằng chương trình Universal Math Solver, không hiện ảnh thì vô thống kê hỏi đáp của mình, ngày 30/5/2020 vào lúc 8:25)

\(=\frac{1}{2}\left[\left(a+b+c\right)^2+6\left(a+b+c\right)+9\right]=\frac{1}{2}\left(a+b+c+3\right)^2\)(**)

Từ (*) và (**) suy ra \(3-K\ge\frac{\left(a+b+c+3\right)^2}{\frac{1}{2}\left(a+b+c+3\right)^2}=2\Rightarrow K\le1\)

Vậy ta có điều phải chứng minh

Đẳng thức xảy ra khi a = b = c = 1

30 tháng 5 2020

Áp dụng BĐT Cô-si,ta có :

\(a^2+1\ge2a\)

\(\Rightarrow\frac{a}{a^2+2b+3}\le\frac{a}{2a+2b+2}=\frac{1}{2}\left(\frac{a}{a+b+1}\right)\)

Tương tự : \(\frac{b}{b^2+2c+3}\le\frac{1}{2}\left(\frac{b}{b+c+1}\right);\frac{c}{c^2+2a+3}\le\frac{1}{2}\left(\frac{c}{c+a+1}\right)\)

\(\Rightarrow\frac{a}{a^2+2b+3}+\frac{b}{b^2+2c+3}+\frac{c}{c^2+2a+3}\le\frac{1}{2}\left(\frac{a}{a+b+1}+\frac{b}{b+c+1}+\frac{c}{c+a+1}\right)\)

Áp dụng BĐT Bu-nhi-a-cốp-ski,ta có :

\(\frac{a}{a+b+1}=\frac{a\left(a+b+c^2\right)}{\left(a+b+1\right)\left(a+b+c^2\right)}\le\frac{a^2+ab+ac^2}{\left(a^2+b^2+c^2\right)^2}=\frac{a^2+ab+ac^2}{9}\)

TT : ...

Cộng lại ta được :

\(\frac{a}{a+b+1}+\frac{b}{b+c+1}+\frac{c}{c+a+1}\le\frac{a^2+ab+ac^2}{9}+\frac{b^2+bc+ba^2}{9}+\frac{c^2+ca+cb^2}{9}\)

\(=\frac{a^2+b^2+c^2+ab+bc+ac+ac^2+ba^2+cb^2}{9}\le\frac{3+3+3}{9}=1\)

\(\Rightarrow\frac{a}{a^2+2b+3}+\frac{b}{b^2+2c+3}+\frac{c}{c^2+2a+3}\le\frac{1}{2}\)

Dấu "=" xảy ra khi a = b = c = 1